What is the slope of 4x + 3y = −24?

Answers

Answer 1

Answer:

slope is 4/3x

Step-by-step explanation:


Related Questions

1/8 divided by 2 3/4

Answers

Answer:

2 and 1/8

Step-by-step explanation:

Reduce fractions where possible.

Then your initial equation becomes 1/8 divied by 1/17.Applying the fractions formula for division, 1 17 over 8 and 1 then multiply those. Then simplfy.

[tex]\frac{1}{8}[/tex] divided by [tex]2\frac{3}{4}[/tex] is equal to [tex]\frac{1}{22}[/tex] we get by converting the mixed fraction to improper and divide them both.

The given expression is [tex]\frac{1}{8}[/tex] divided by [tex]2\frac{3}{4}[/tex].

Convert the mixed number [tex]2\frac{3}{4}[/tex] to an improper fraction.

[tex]2\frac{3}{4} = \frac{2 \times 4 + 3}{4}[/tex]

[tex]= \frac{11}{4}[/tex]

Now, we can rewrite the expression as:

[tex]\frac{1}{8} \div \frac{11}{4}[/tex]

To divide fractions, we multiply the first fraction by the reciprocal of the second fraction:

[tex]\frac{1}{8} \times \frac{4}{11}[/tex]

Next, let's simplify the multiplication:

[tex]\frac{1 \times 4}{8 \times 11} = \frac{4}{88}[/tex]

We can further simplify the fraction by dividing the numerator and denominator by their greatest common divisor, which is 4:

[tex]\frac{4}{88} = \frac{1}{22}[/tex]

Therefore,[tex]\frac{1}{8}[/tex] divided by [tex]2\frac{3}{4}[/tex] is equal to [tex]\frac{1}{22}[/tex].

To learn more on Fractions click:

https://brainly.com/question/10354322

#SPJ6

If 2a + 3 < 10, then which of these could be a value of a?

a. 6 b. 4 c. 2 d. 10

Answers

Answer:

it's going to be d.

Answer: c

Step-by-step explanation:

2*6=12

12+3=15

15 is not less than 10

2*4=8

8+3=11

11 is not less than 10

2*2=4

4+3=7

7 is less than 10

10*2=20

20+3=23

23 is not less than 10

A restaurant menu shows the price for 2 chicken wings is $2.30 and the price for 6 chicken wings is $6.90. The function below gives the price, y, for x wing
y - 2.30 = 1.15(x - 2)
If this function is graphed in the coordinate plane, what is the y-intercept?

Answers

Answer:

y - intercept = 0 [(0,0)]

Step-by-step explanation:

Given the equation y - 2.30 = 1.15(x-2), the y-intercept is when x is 0.

So to find your answer, just substitute in 0 for x, and solve for y:

y - 2.30 = 1.15(x - 2) →

x = 0

y - 2.30 = 1.15(0 - 2) →

y - 2.30 = 1.15(-2)

y - 2.30 = -2.30 →

+2.30 +2.30

y = 0

The freshman spirit club took a trip to the state fair .There were 59 students and 6 chaperones, and the total admission cost for the group was 508$.Student tickets cost 2$ more than the chaperone tickets.Write an equation to represent the cost of a student ticket

Answers

Answer:

Step-by-step explanation:

First you determine the variables x and y as:

x: the value for student tickets

y: the value for chaperone tickets.

Knowing that student tickets cost $ 2 more than companion tickets, it is represented by the equation:

x= y +2

On the other hand, there were 59 students and 6 companions. And the total cost of admission for the group was $ 508, that is to say that what all the students and the companions have paid adds up to $ 508. Expressed by an equation:

59x + 6y = 508

Then the system of equations to solve and thus obtain the price of a student ticket and the price of a companion ticket is:

[tex]\left \{ {{x=y+2} \atop {59x+6y=508}} \right.[/tex]

Solving for x, the price of a student ticket:

Rearranging  the first equation,

y = x - 2

Replacing in the second equation and solving for x:

59*x + 6*(x -2)=508

59*x + 6*x -12=508

65*x -12=508

65*x= 508 +12

65*x= 520

[tex]x=\frac{520}{65}[/tex]

x=8

The cost of a student ticket is 8$

Enter an equation to describe the linear relationship.


A bamboo reed is planted when it is 15 centimeters tall. It grows 3.2 centimeters per week. Let y represent the total growth and x for the number of weeks.
The equation is y =

Answers

Answer:

The equation is y = 3.2x + 15

Step-by-step explanation:

We know that the beginning height is 15 cm, so that will be our y-intercept, since we started with that. We also need the slope, which is the rate of change. The rate of change is 3.2 cm per week, so the slope is 3.2. This makes the equation y = 3.2x + 15. I hope this helps, and I hope it's not wrong, which I don't think it is.

Please help me I’m from Texas so I am stupid

Answers

Answer:

Given h(x) = -x + 2, find h(4).

Step-by-step explanation:

Answer:

Graph A

Step-by-step explanation:

Because the inequality is

[tex] \leqslant or \geqslant [/tex]

it means the line will be solid, not dashed. Also, because the y variable is on the less than side of the equation, it means the shaded are will be under the line.

Estimate the sum 78.5+51.4

Answers

The answer to that question is 129.9. Hope this helps and if you could can you mark as brainliest? I’m trying to rank up and I just need 2 more!

Answer:

129.9

Step-by-step explanation:

78.5

+ 51.4

129.9

Hope this helps! (:

Amber is making a scale drawing of a movie screen. The actual dimisions of a screen in her local theater are 60 ft wide and 20 ft tall. In her scale drawing, the screen has a width of 9 in. What will the height of the screen on her drawing be? a 2 inches b 6 inches c 3 inches d 10 inches

Answers

Answer:

c 3 inches

Step-by-step explanation:

Given the dimension in feet

Width = 60 ft

Height = 20 ft

Given the dimension in inches

Width = 9in

Height = ?

To get the height in inches, we will use the equality postulate:

60 ft = 9in

20ft = x

Cross multiply

60x = 9*20

60x =180

x = 180/60

x = 3in

Hence the height of the screen will be 3inches

Mr. Swanson bought a package of 10 disposable razors for $6.30.He found that each razor lasted for 1 week. What was the cost per day?

Answers

Answer:

don't even know hope u get it tho

Kelsey buys several pairs of uniform pants for $17.95 each, and a sweater for $24. Jeana shops at a different store and buys several pairs of uniform pants for $18.95 each, plus a sweater for $18. They set up the situation with the equation below, where x is the number of pairs of pants. Is there a situation in which they pay the same amount for their purchases? Which statements are true? Select all that apply.Kelsey buys several pairs of uniform pants for $17.95 each, and a sweater for $24. Jeana shops at a different store and buys several pairs of uniform pants for $18.95 each, plus a sweater for $18. They set up the situation with the equation below, where x is the number of pairs of pants. Is there a situation in which they pay the same amount for their purchases? Which statements are true? Select all that apply.

Answers

Answer:

If both Kelsey and Jeana purchase 6 pairs of uniform pants then they would pay the same amount for their purchases.

Step-by-step explanation:

The information provided is as follows:

Kelsey buys several pairs of uniform pants for $17.95 each, and a sweater for $24. Jeana shops at a different store and buys several pairs of uniform pants for $18.95 each, plus a sweater for $18.

The variable x is the number of pairs of pants.

The total cost function for Kelsey will be:

[tex]TC_{K}=17.95\cdot x+24[/tex]

The total cost function for Jeana will be:

[tex]TC_{J}=18.95\cdot x+18[/tex]

Consider that both pay the same total cost for their purchases.

Compute the value of x as follows:

            [tex]TC_{K}=TC_{J}[/tex]

[tex]17.95\cdot x+24=18.95\cdot x+18\\\\18.95\cdot x-17.95\cdot x=24-18\\\\x=6[/tex]

Thus, if both Kelsey and Jeana purchase 6 pairs of uniform pants then they would pay the same amount for their purchases.

A rectangle has a length of 64 cm. The rectangle is reduced by a scale factor of 3/4. What is the length of the reduced rectangle?

Answers

Answer:

48 cm

Step-by-step explanation:

64 divided by 4 is 16. multiply 16 by 3 and the answer is 48.

Kristy rides 6 miles on her bicycle in 40 minutes. Jax rides 4 miles on his bicycle in 30 minutes. Who is faster? Remember to support your answer with a mathematical statement!

Answers

Answer: Kristy

Step-by-step explanation: 6x=40 4x=30, 40/6 is 6.6 repeating which means she travels a mile in 6.6 minutes and 30/4 is 7.5. 6.6 repeating is lower than 7.5 which means it is faster.

Find the rate of change in the graph

Answers

Answer:

4/3

hope this helped :)

Find the slope of the line passing through each of the following pairs of points and draw the graph of the line. (-2,3), (5,5)

Answers

Answer: M= 2/7

Step-by-step explanation:

Use the graph to find the midpoint between O and H.
(4, -1)


(-2, 8)


(8, -2)


(-1, 4)

Answers

Answer:

(-1, 4)

Step-by-step explanation:

-5+3/2 = -1

6+ 2/2 = 4

(-1, 4)

What is the 6th term of the geometric progression
12.6.3, ... ?

Answers

Answer:

a₆ = [tex]\frac{3}{8}[/tex]

Step-by-step explanation:

The n th term of a geometric progression is

[tex]a_{n}[/tex] = a₁ [tex]r^{n-1}[/tex]

where a₁ is the first term and r the common ratio

Here a₁ = 12 and r = [tex]\frac{a_{2} }{a_{1} }[/tex] = [tex]\frac{6}{12}[/tex] = [tex]\frac{1}{2}[/tex] , thus

a₆ = 12[tex](\frac{1}{2}) ^{5}[/tex] = 12 × [tex]\frac{1}{32}[/tex] = [tex]\frac{12}{32}[/tex] = [tex]\frac{3}{8}[/tex]


If a population roughly doubles in the course of 22 years, its growth rate would be close to
C) 2.0
A) 1.5
D) 3.2
B) 1.2

Answers

Answer:

hmm you need help

Step-by-step explanation:

Is this a function or no

Answers

Answer: Yes it is a function

Step-by-step explanation:

Answer:

YES

Step-by-step explanation:

PLEASE ANSWER ASAP WILL MARK BRAINLEST
Paul and Dana both want to find the change from the 6 AM temperature to the new 12 PM temperature.

Paul says that you can use a number line and add the distance from 0 to the 6 AM temperature and from 0 to the new 12 PM temperature. Dana says that you can subtract -1.4 from the new 12 PM temperature.

Is either Paul or Dana correct> Explain your reasoning and find the change in temperature.

Answers

Answer:

Paul is correct

Step-by-step explanation:

You can add the distance of 0 to 6 AM and 0 to 12 PM, and doing so on a number line. On a number line you would add them and lets say 6 AM is 6 degrees and 12 PM is 12 degrees, you would add 6 and 12 to get 18. While doing so you would got 1 2 3 4 5 6 7 8 9 and so on. Thus, finding the difference.

I know I'm late but can I still get that brainliest

What number should be subtracted from 3/7 to get 5/7

Answers

8/7 because you can just add the two numbers together and that’s ur answer

Help!! Anyone know how to do this?

Answers

Answer:

42.5

Step-by-step explanation:

Answer:

x = 37.5

Step-by-step explanation:

First, you'll have to find the amount between the similar triangles. This is what  I mean:

42.5/17 = 2.5

20/8 = 2.5

So, that means that 15(2.5) = x.

x = 37.5

If your UBER and your bill is 20$ they will ask if you want to leave a 4$ tip. what percent tip would that be?​

Answers

That is a 20% tip out of $20

4 out of 20 is 20% so they left a 20% percent tip

Jason and Henry go to the movie theater and purchase refreshments for their friends. Jason spends a total of $66.75 on 12 drinks and 1 bag of popcorn. Henry spends a total of $82.50 on 3 drinks and 10 bags of popcorn. Write a system of equations that can be used to find the price of one drink and the price of one bag of popcorn. Using these equations, determine and state the price of popcorn, to the nearest cent.

Answers

Answer:

The cost of one drink is $5

Step-by-step explanation:

3x-120x=82.50-667.5

120x-3x=667.50-82.50

117x=585

x=5

An animal shelter has $2500 in its reserve fund. The shelter charges $40 per animal placement and would like to have at least $4000 in its reserve fund. Write an inequality to represent this situation.

Answers

Answer:

Step 1

Write and solve the inequality:

2,500 + 40a ≥ 4,000, or 40a ≥ 1,500

a ≥ 37.5

Step 2

If the shelter places 30 cats and 10 dogs,

or 40 animals, that will be enough to meet

its goal, because a = 40 is a solution to the

inequality a ≥ 37.5.

Solve for s. 7 - 45 = -3s ​

Answers

Answer:

12.7=s

Step-by-step explanation:

First, write the equation and combine like terms.

7-45=-3s

-38=-3s

Next, divide both sides by 3.

-38=-3s

/-3  /-3

12.7=s (I rounded the answer.)

Hope this helps! Have a great day C:

Kelly gets a Sonic drink everyday. If she uses her app she gets the drinks for 1/2
off the price of the drinks. If the drinks are normally $1.88 which equation could
be used to find y (the cost) of x (number of drinks) she purchases if she uses her
app?

Answers

Answer:

6.58

Step-by-step explanation:

divide 1.88 by 2 to get .94 because he gets her drinks half off. Next, multiply by 7 because there are seven days in a week. she will spend 6.58 a week on drinks.

Answer:

1.88/2  0.94x7 for one week Kelly will pay $6.58 for a sonic drink

the answer is B

Step-by-step explanation:

1.88/2= 0.94

0.94x7=6.58

for one week Kelly will pay $6.58 for a sonic drink

hope this helps:) have a good day :)

Given 5x - y = 16, solve for y

Answers

i don’t know if this is helpful but i tried , sorry

y = 5x -16 will be the Solution for y in terms of x for the given equation.

What is Linear function?

A linear function in mathematics is one that has either one or two variables and no exponents. It is a function with a straight line as its graph.

Given a linear function 5x - y = 16

Solution for y in terms of x will be

5x - y = 16

=> 5x = 16 +y

=> y = 5x -16

Therefore, the Solution for y in terms of x for the given equation will be:

y = 5x -16.

Learn more about Linear Functions here:

https://brainly.com/question/20286983

#SPJ2

Find the slope of a line PERPENDICULAR TO LM:
L(3,-5), M(-4, -1)

Answers

Answer:

[tex]\frac{7}4[/tex]

Step-by-step explanation:

First, we need to find the slope, which is defined as the change in y over the change in x:

[tex]\frac{-5-(-1)}{3-(-4)} = \frac{-4}{7}[/tex]

To find a slope that's perpendicular to that, we just take the negative reciprocal of that slope, which is [tex]\frac{7}4[/tex].

I hope this helped.

The function g(x) = (x + 2)2 + 4 is a transformation of the parent function f(x) = x2. Select all the statements that are true about the transformation.
A. Function g is the result of f being translated right 2 units and down 4 units.
B. Function g is the result of f being translated left 2 units and up 4 units.
C. The graph of function f opens upward.
D. The graph of function g opens downward.
E. The graph of function f is compressed by a factor of 2.

Answers

Answer:

Step-by-step explanation:

B

C

Using translation and parabola concepts, it is found that the correct options are:

A. Function g is the result of f being translated right 2 units and down 4 units.

C. The graph of function f opens upward.

------------------------------

The parent function is: [tex]f(x) = x^2[/tex]Function g is given by: [tex]g(x) = (x + 2)^2 + 4[/tex]Shifting a function f(x) a units to the right is the same as finding f(x + a).Shifting a function f(x) a units up is the same as finding f(x) + a.From the last two bullet points, option A is correct and option B is incorrect.In both functions, f and g, the term that multiplies [tex]x^2[/tex] is positive, thus, both open upward, which means that option C is correct and option D is incorrect.Both are multiplied by the same factor, 1, thus none are compressed, and option E is incorrect.

A similar problem is given at https://brainly.com/question/4521517

How did you find each square root? ​

Answers

Answer:

......

Step-by-step explanation:

join on zoom meeting now I'd 4989249097 and password MsS8pt join now for fun only girls

Other Questions
Find the value of [tex]\frac{a^{2} }{a^{4}+a^{2}+1}[/tex] if [tex]\frac{a}{a^{2}+a+1}[/tex] = [tex]\frac{1}{6}[/tex]. Please show all work. Carlos was baking 1,000 cookies for a yard sale. 3 people bought 82 cookies each. How many cookies does Carlos have left? Explain. By the time the Europeans arrived in North America, Indians: Casey bought 3/8 pound of plums and 5/12 pound of bananas. How many pounds of fruit did he buy? If a persons religious beliefs conflict with the law or lead to bias against other groups, should the government protect the exercise of those beliefs? Why or why not? Who helped Heran Cortes and his soldiers defeat Aztec 6 apples cost exactly the same as 9 oranges. 10 apples and 10 oranges cost $7.50. Find the cost of 1 apple and the cost of 1 orange. Look at the Electoral map of the 2016 election below. Do you expect your state to vote the same in the upcoming election? Why or why not? *Pleaseee pleaseee help Marty stabs Chriss hand while playing poker because he is mad he is losing. Ordinarily, a person could just wrap a wound like this and be fine. However, Chris has hemophilia, but he hasnt told anyone even though hes had it for years, which means he bleeds more than most people and can die from even minor injuries. He bleeds to death.CAN MARTY BE CHARGED FOR THE DEATH OF CHRIS? two plus two plus four?? On Monday, you run treadmill for 1/2 hour at x miles per hour. On Tuesday, you walk the same distance on the treadmill, at 2 mph slower, and it takes you 3/4 hour.How many miles did you run on the treadmill on Monday? Anne works for a company that manufactures liquid dyes for clothing. She currently wants to mix up some 30%-concentrated red dye. She has 66 gallons of 21%-concentrated red dye, as well as plenty of 51%-concentrated red dye. How many gallons of the 51%-concentrated red dye will Anne need to add to the 21%-concentrated red dye to make a batch with a concentration of 30%? What is the ratio of the two quantities? Warning: different units! a 4 seconds to 2 min What are the strengths of 3rd person point of view? what is the least common factor of 25 and 8 Read "The Workers of Ellis Island"; and look at the photo. Three men standing behind a seated man. The first man is wearing all black with a white-collar. The second man is wearing a white shirt with a black jacket. The third man is wearing all white. Which statement is most likely accurate? The man on the left works as a surgeon at Ellis Island. The man who is standing in the center is a clerk at Ellis Island. The man who is sitting recently received medical attention. The man on the right has traveled from a foreign country. All of the following experience a widespread spatial distribution due to colonialism and imperialism EXCEPTA. EnglishB. Christianity C. SpanishD. FrenchE. Judaism DO NOT ANSWER IF YOU DO NOT KNOW, Will give brainliest, like, comment, and 5 points to answer Each of these is a pair of equivalent ratios, for each pair explain why they are equivalent. 5:1 and 15:310.2 and 25.5198:1,287 and 2:13 Solve for m: 2m + 5-3m = 7 Which one? A or B?